Những câu hỏi liên quan
An Vy
Xem chi tiết
Lê Minh Đức
Xem chi tiết
Akai Haruma
9 tháng 10 2017 lúc 17:28

Ta có \(a^2b^2+b^2c^2+c^2a^2\geq a^2b^2c^2\Leftrightarrow \frac{1}{a^2}+\frac{1}{b^2}+\frac{1}{c^2}\geq 1\)

BĐT cần chứng minh tương đương với \(\frac{\frac{1}{c^3}}{\frac{1}{a^2}+\frac{1}{b^2}}+\frac{\frac{1}{b^3}}{\frac{1}{a^2}+\frac{1}{c^2}}+\frac{\frac{1}{a^3}}{\frac{1}{b^2}+\frac{1}{c^2}}\geq \frac{\sqrt{3}}{2}\)

Đặt \((\frac{1}{a},\frac{1}{b},\frac{1}{c})=(x,y,z)\). Bài toán trở thành: 

Cho \(x,y,z>0|x^2+y^2+z^2\geq 1\). CMR \(P=\frac{x^3}{y^2+z^2}+\frac{y^3}{z^2+x^2}+\frac{z^3}{x^2+y^2}\geq \frac{\sqrt{3}}{2}\)

Lời giải:

 Áp dụng BĐT Cauchy -Schwarz:

\(P=\frac{x^4}{xy^2+xz^2}+\frac{y^4}{yz^2+yx^2}+\frac{z^4}{zx^2+zy^2}\geq \frac{(x^2+y^2+^2)^2}{x^2(y+z)+y^2(x+z)+z^2(x+y)}\) (1)

Không mất tính tổng quát, giả sử \(x\geq y\geq z\Rightarrow x^2\geq y^2\geq z^2\) 

Và \(y+z\leq z+x\leq x+y\). Khi đó, áp dụng BĐT Chebyshev: 

\(3[x^2(y+z)+y^2(x+z)+z^2(x+y)]\leq (x^2+y^2+z^2)(y+z+x+z+x+y)\)

\(\Leftrightarrow x^2(y+z)+y^2(x+z)+z^2(x+y)\leq \frac{2(x^2+y^2+z^2)(x+y+z)}{3}\)

Theo hệ quả của BĐT Am-Gm thì: \((x+y+z)^2\leq 3(x^2+y^2+z^2)\Rightarrow x+y+z\leq \sqrt{3(x^2+y^2+z^2)}\)

\(\Rightarrow x^2(y+z)+y^2(x+z)+z^2(x+y)\leq \frac{2(x^2+y^2+z^2)\sqrt{3(x^2+y^2+z^2)}}{3}\) (2)

Từ (1),(2) suy ra \(P\geq \frac{3(x^2+y^2+z^2)^2}{2(x^2+y^2+z^2)\sqrt{3(x^2+y^2+z^2)}}=\frac{\sqrt{3(x^2+y^2+z^2)}}{2}\geq \frac{\sqrt{3}}{2}\)

Ta có đpcm

Dáu bằng xảy ra khi \(x=y=z=\frac{1}{\sqrt{3}}\Leftrightarrow a=b=c=\sqrt{3}\)

Kiệt Nguyễn
5 tháng 5 2020 lúc 12:58

Đặt \(x=\frac{1}{a};y=\frac{1}{b};z=\frac{1}{c}\)

Khi đó giả thiết được viết lại là \(x^2+y^2+z^2\ge1\)và ta cần chứng minh \(\frac{x^3}{y^2+z^2}+\frac{y^3}{z^2+x^2}+\frac{z^3}{x^2+y^2}\ge\frac{\sqrt{3}}{2}\)(*)

Áp dụng BĐT Bunhiacopxki dạng phân thức, ta được:

\(VT_{\left(^∗\right)}=\frac{x^4}{x\left(y^2+z^2\right)}+\frac{y^4}{y\left(z^2+x^2\right)}+\frac{z^4}{z\left(x^2+y^2\right)}\)\(\ge\frac{\left(x^2+y^2+z^2\right)^2}{x\left(y^2+z^2\right)+y\left(z^2+x^2\right)+z\left(x^2+y^2\right)}\)

Đến đây ta đi chứng minh \(\frac{\left(x^2+y^2+z^2\right)^2}{x\left(y^2+z^2\right)+y\left(z^2+x^2\right)+z\left(x^2+y^2\right)}\ge\frac{\sqrt{3}}{2}\)

\(\Leftrightarrow2\left(x^2+y^2+z^2\right)^2\)\(\ge\sqrt{3}\left[x\left(y^2+z^2\right)+y\left(z^2+x^2\right)+z\left(x^2+y^2\right)\right]\)

Ta có: \(x\left(y^2+z^2\right)=\frac{1}{\sqrt{2}}\sqrt{2x^2\left(y^2+z^2\right)\left(y^2+z^2\right)}\)\(\le\frac{1}{\sqrt{2}}\sqrt{\left(\frac{2x^2+y^2+z^2+y^2+z^2}{3}\right)^3}\)

\(=\frac{2\sqrt{3}}{9}\left(x^2+y^2+z^2\right)\sqrt{x^2+y^2+z^2}\)

Tương tự ta có: \(y\left(z^2+x^2\right)\le\frac{2\sqrt{3}}{9}\left(x^2+y^2+z^2\right)\sqrt{x^2+y^2+z^2}\)

\(z\left(x^2+y^2\right)\le\frac{2\sqrt{3}}{9}\left(x^2+y^2+z^2\right)\sqrt{x^2+y^2+z^2}\)

Cộng theo vế của 3 BĐT trên, ta được: 

\(\text{∑}_{cyc}\left[x\left(y^2+z^2\right)\right]\le\frac{2\sqrt{3}}{3}\left(x^2+y^2+z^2\right)\sqrt{x^2+y^2+z^2}\)

\(\Leftrightarrow\sqrt{3}\text{∑}_{cyc}\left[x\left(y^2+z^2\right)\right]\le2\left(x^2+y^2+z^2\right)\sqrt{x^2+y^2+z^2}\)

Cuối cùng ta cần chứng minh được

\(2\left(x^2+y^2+z^2\right)\sqrt{x^2+y^2+z^2}\le2\left(x^2+y^2+z^2\right)^2\)

\(\Leftrightarrow x^2+y^2+z^2\ge1\)(đúng)

Đẳng thức xảy ra khi \(x=y=z=\frac{1}{\sqrt{3}}\Rightarrow a=b=c=\sqrt{3}\)

Khách vãng lai đã xóa
Cầm Dương
Xem chi tiết
Thắng Nguyễn
8 tháng 5 2018 lúc 16:50

Đề cần bổ sung \(a,b>0\) nhé

\(BDT\Leftrightarrow\frac{\left(a-b\right)^2\left(a^2+6ab+7b^2\right)}{4b^2\left(a+b\right)^2\left(a+2b\right)}\ge0\) *luôn đúng*

\("="\Leftrightarrow a=b\)

An Vy
Xem chi tiết
Blue Moon
Xem chi tiết
alibaba nguyễn
22 tháng 2 2019 lúc 8:38

Đặt \(\left(\frac{1}{a};\frac{1}{b};\frac{1}{c}\right)=\left(x;y;z\right)\)thì bài toán thành

\(x+y+z=2\) chứng minh rằng

\(\frac{x^3}{\left(2-x\right)^2}+\frac{y^3}{\left(2-y\right)^2}+\frac{z^3}{\left(2-z\right)^2}\ge\frac{1}{2}\)

Trước hết ta chứng minh:

Ta có: \(\frac{x^3}{\left(2-x\right)^2}+\frac{2-x}{8}+\frac{2-x}{8}\ge\frac{3x}{4}\)

\(\Leftrightarrow\frac{x^3}{\left(2-x\right)^2}\ge x-\frac{1}{2}\)

\(\Rightarrow VP\ge\left(x+y+z\right)-\frac{3}{2}=2-\frac{3}{2}=\frac{1}{2}\)

Ai Ai Ai
Xem chi tiết
Kiệt Nguyễn
Xem chi tiết
Tran Le Khanh Linh
30 tháng 4 2020 lúc 21:02

\(a^2b^2c^2+\left(a+1\right)\left(1+b\right)\left(1+c\right)\ge a+b+c+ab+bc+ca+3\)

\(\Leftrightarrow\left(abc\right)^2+abc-2\ge0\Leftrightarrow\left(abc+2\right)\left(abc-1\right)\ge0\Leftrightarrow abc\ge1\)

Áp dụng BĐT Cosi ta có:

\(\frac{a^3}{\left(b+2c\right)\left(2c+3a\right)}+\frac{b+2c}{45}+\frac{2c+3a}{75}\ge3\sqrt[3]{\frac{a^3}{\left(b+2c\right)\left(2c+3b\right)}\cdot\frac{b+2c}{45}\cdot\frac{2c+3a}{75}}=\frac{a}{5}\left(1\right)\)

Tương tự ta có: \(\hept{\begin{cases}\frac{b^3}{\left(c+2a\right)\left(2a+3b\right)}+\frac{c+2a}{45}+\frac{2a+3b}{75}\ge\frac{b}{5}\left(2\right)\\\frac{c^3}{\left(a+2b\right)\left(2b+3c\right)}+\frac{a+2b}{45}+\frac{2b+3c}{75}\ge\frac{c}{5}\left(3\right)\end{cases}}\)

Từ (1)(2)(3) ta có:

\(P+\frac{2\left(a+b+c\right)}{15}\ge\frac{a+b+c}{5}\Leftrightarrow P\ge\frac{1}{15}\left(a+b+c\right)\)

Mà \(a+b+c\ge3\sqrt[3]{abc}\Rightarrow S\ge\frac{1}{5}\)

Dấu "=" xảy ra <=> a=b=c=1

Khách vãng lai đã xóa
Kamado Tanjiro
3 tháng 5 2020 lúc 7:25

CHÚC BAN HỌC GIỎI

Khách vãng lai đã xóa
Phạm Bảo Nam
4 tháng 5 2020 lúc 16:20

đây\(x = {-b \pm \sqrt{b^2-4ac} \over 2a}\)

Khách vãng lai đã xóa
Giao Khánh Linh
Xem chi tiết
Phạm Bá Tâm
Xem chi tiết
Nguyễn Đăng Nhân
9 tháng 2 2022 lúc 16:24

\(\frac{bc}{a^2\left(b+c\right)}+\frac{ca}{b^2\left(c+a\right)}+\frac{ab}{c^2\left(a+b\right)}\ge\frac{1}{2a}+\frac{1}{2b}+\frac{1}{2c}\)

\(\Rightarrow\frac{bc}{a^2\left(b+c\right)}+\frac{b+c}{4bc}\ge2\sqrt{\frac{bc}{a^2\left(b+c\right)}\cdot\frac{b+c}{4bc}}=\frac{1}{a}\)

\(\Rightarrow\frac{ca}{b^2\left(c+a\right)}+\frac{c+a}{4ca}\ge2\sqrt{\frac{ca}{b^2\left(c+a\right)}\cdot\frac{c+a}{4ca}}=\frac{1}{b}\)

\(\Rightarrow\frac{ab}{c^2\left(a+b\right)}+\frac{a+b}{4ab}\ge2\sqrt{\frac{ab}{c^2\left(a+b\right)}\cdot\frac{a+b}{4ab}}=\frac{1}{c}\)

Cộng theo vế các bất đẳng thức trên ta được:

\(\frac{bc}{a^2\left(b+c\right)}+\frac{ca}{b^2\left(c+a\right)}+\frac{ab}{c^2\left(a+b\right)}+\frac{b+c}{4bc}+\frac{c+a}{4ca}+\frac{a+b}{4ab}\ge\frac{1}{a}+\frac{1}{b}+\frac{1}{c}\)

\(\frac{b+c}{4bc}+\frac{c+a}{4ca}+\frac{a+b}{4ab}=\frac{1}{2}\left(\frac{1}{a}+\frac{1}{b}+\frac{1}{c}\right)\)nên:

\(\frac{bc}{a^2\left(b+c\right)}+\frac{ca}{b^2\left(c+a\right)}+\frac{ab}{c^2\left(a+b\right)}\ge\frac{1}{a}+\frac{1}{b}+\frac{1}{c}-\frac{1}{2}\left(\frac{1}{a}+\frac{1}{b}+\frac{1}{c}\right)\) 

hay\(\frac{bc}{a^2\left(b+c\right)}+\frac{ca}{b^2\left(c+a\right)}+\frac{ab}{c^2\left(a+b\right)}\ge\frac{1}{2a}+\frac{1}{2b}+\frac{1}{2c}\)

Bất đẳng thức xảy ra khi \(a=b=c\)

Khách vãng lai đã xóa
Lưu Nguyễn Hà An
10 tháng 2 2022 lúc 10:40

bạn giỏi quá

Nguyễn Đăng Nhân

Khách vãng lai đã xóa